38
COMPILATION OF CASE DIGEST CONSTI 2: LIBERTY OF ABODE AND TRAVEL AND FREEDOM OF ASSOCIATION ISV Notes LIBERTY OF ABODE 1. SALONGA V. HERMOSO 97 SCRA 121 – Political Law – Right to Travel Even During Martial Law FACTS: This is not the first time Jovito R. Salonga came to the Supreme Court by way of a mandamus proceeding to compel the issuance to him of a certificate of eligibility to travel. In the first case, Salonga v. Madella (GR L-49130), the case became moot and academic as the Office of the Solicitor General, in its answer to the petition, stated that the travel eligibility certificate was not denied and, as a matter of fact, had been granted . Herein, in the motion to dismiss of the Solicitor General dated 21 April 1980, it was stated that the certificate of eligibility to travel had been granted Salonga. A xeroxed copy was enclosed. ISSUE: Whether or not the right to travel may be prohibited during martial law. HELD: The Travel Processing Center should exercise the utmost care to avoid the impression that certain citizens desirous of exercising their constitutional right to travel could be subjected to inconvenience or annoyance. In the address of President and Prime Minister Ferdinand E. Marcos before the American Newspaper Publishers Association on 22 April 1980, he emphasized anew the respect accorded constitutional rights. The freedom to travel is certainly one of the most cherished. He cited with approval the ringing affirmation of Willoughby, who, as he noted was "partial to the claims of liberty." Burdick and Willis, both of whom were equally convinced that there be no erosion to human rights even in times of martial law, likewise received from President Marcos the accolade of his approval. It would appear, therefore, that in case of doubt of the Officer-in-Charge of the Travel Processing Center, the view of General Fabian Ver should immediately be sought. It goes without saying that the petition for such certificate of eligibility to travel be filed at the earliest opportunity to facilitate the granting thereof and preclude any disclaimer as to the person desiring to travel being in any way responsible for any delay. 2. MARCOS VS. MANGLAPUS [GR 88211, 15 September 1989] En Banc, Cortes (J): 6 concur, 1 concurs in separate opinion, 5 dissent in separate opinions, 1 on leave FACTS: In February 1986, Ferdinand E. Marcos was deposed from the presidency via the non- violent "people power" revolution and forced into exile. In his stead, Corazon C. Aquino was declared President of the Republic under a revolutionary government. Her ascension to and consolidation of power have not been unchallenged. The failed Manila Hotel coup in 1986 led by political leaders of Mr. Marcos, the takeover of television station Channel 7 by rebel troops led by Col. Canlas with the support of "Marcos loyalists" and the unsuccessful plot of the Marcos spouses to surreptitiously return from Hawaii with mercenaries aboard an aircraft chartered by a Lebanese arms dealer awakened the nation to the capacity of the Marcoses to stir trouble even from afar and to the fanaticism and blind loyalty of their followers in the country. The

Liberty of Abode and Travel to Freedom of Association

Embed Size (px)

DESCRIPTION

DIGESTS

Citation preview

Page 1: Liberty of Abode and Travel to Freedom of Association

COMPILATION OF CASE DIGEST CONSTI 2: LIBERTY OF ABODE AND TRAVEL AND FREEDOM OF ASSOCIATION ISV Notes

LIBERTY OF ABODE

1.SALONGA V. HERMOSO

97 SCRA 121 – Political Law – Right to Travel Even During Martial Law

FACTS: This is not the first time Jovito R. Salonga came to the Supreme Court by way of a mandamus proceeding to compel the issuance to him of a certificate of eligibility to travel. In the first case, Salonga v. Madella (GR L-49130), the case became moot and academic as the Office of the Solicitor General, in its answer to the petition, stated that the travel eligibility certificate was not denied and, as a matter of fact, had been granted. Herein, in the motion to dismiss of the Solicitor General dated 21 April 1980, it was stated that the certificate of eligibility to travel had been granted Salonga. A xeroxed copy was enclosed.

ISSUE: Whether or not the right to travel may be prohibited during martial law.

HELD: The Travel Processing Center should exercise the utmost care to avoid the impression that certain citizens desirous of exercising their constitutional right to travel could be subjected to inconvenience or annoyance. In the address of President and Prime Minister Ferdinand E. Marcos before the American Newspaper Publishers Association on 22 April 1980, he emphasized anew the respect accorded constitutional rights. The freedom to travel is certainly one of the most cherished. He cited with approval the ringing affirmation of Willoughby, who, as he noted was "partial to the claims of liberty." Burdick and Willis, both of whom were equally convinced that there be no erosion to human rights even in times of martial law, likewise received from President Marcos the accolade of his approval. It would appear, therefore, that in case of doubt of the Officer-in-Charge of the Travel Processing Center, the view of General Fabian Ver should immediately be sought. It goes without saying that the petition for such certificate of eligibility to travel be filed at theearliest opportunity to facilitate the granting thereof and preclude any disclaimer as to the person desiring to travel being in any way responsible for any delay.

2. MARCOS VS. MANGLAPUS [GR 88211, 15 September 1989]

En Banc, Cortes (J): 6 concur, 1 concurs in separate opinion, 5 dissent in separate opinions, 1 on leave

FACTS: In February 1986, Ferdinand E. Marcos was deposed from the presidency via the non-violent "peoplepower" revolution and forced into exile. In his stead, Corazon C. Aquino was declared President of theRepublic under a revolutionary government. Her ascension to and consolidation of power have not beenunchallenged. The failed Manila Hotel coup in 1986 led by political leaders of Mr. Marcos, the takeover oftelevision station Channel 7 by rebel troops led by Col. Canlas with the support of "Marcos loyalists" and theunsuccessful plot of the Marcos spouses to surreptitiously return from Hawaii with mercenaries aboard anaircraft chartered by a Lebanese arms dealer awakened the nation to the capacity of the Marcoses to stirtrouble even from afar and to the fanaticism and blind loyalty of their followers in the country. Theratification of the 1987 Constitution enshrined the victory of "people power" and also clearly reinforced theconstitutional moorings of Mrs. Aquino's presidency. This did not, however, stop bloody challenges to thegovernment. On 28 August 1987, Col. Gregorio Honasan, one of the major players in the FebruaryRevolution, led a failed coup that left scores of people, both combatants and civilians, dead. There wereseveral other armed sorties of lesser significance, but the message they conveyed was the same — a split inthe ranks of the military establishment that threatened civilian supremacy over the military and brought to thefore the realization that civilian government could be at the mercy of a fractious military. But the armedthreats to the Government were not only found in misguided elements in the military establishment andamong rabid followers of Mr. Marcos. There were also the communist insurgency and the secessionistmovement in Mindanao which gained ground during the rule of Mr. Marcos, to the extent that the communistshave set up a parallel government of their own in the areas they effectively control while the separatists arevirtually free to move about in armed bands. There has been no let up in these groups' determination to wrestpower from the government. Not only through resort to arms but also through the use of propaganda havethey been successful in creating chaos and destabilizing the country. Nor are the woes of the Republic purelypolitical. The accumulated foreign debt and the plunder of the nation attributed to Mr. Marcos and his croniesleft the economy devastated. The efforts at economic recovery, three years after Mrs. Aquino assumed office,have yet to show concrete results in alleviating the poverty of the masses, while the recovery of the ill-gottenwealth of the Marcoses has remained elusive. Now, Mr. Marcos, in his deathbed, has signified his wish toreturn to the Philippines to die. But Mrs. Aquino, considering the dire consequences to the nation of his returnat a time when the stability of government is threatened from various directions and the economy is just

Page 2: Liberty of Abode and Travel to Freedom of Association

COMPILATION OF CASE DIGEST CONSTI 2: LIBERTY OF ABODE AND TRAVEL AND FREEDOM OF ASSOCIATION ISV Notes beginning to rise and move forward, has stood firmly on the decision to bar the return of Mr. Marcos and hisfamily. Hence, Marcos' petition for mandamus and prohibition, asking the court to order Raul Manglapus asSecretary of Foreign Affairs, Catalino Macaraig as Executive Secretary, Sedfrey Ordonez as Secretary ofJustice, Miriam Defensor Santiago as Immigration Commissioner, Fidel Ramos as Secretary of NationalDefense, and Renato de Villa as Chief of Staff, to issue travel documents to Mr. Marcos and the immediatemembers of his family and to enjoin the implementation of the President's decision to bar their return to thePhilippines.

ISSUE:Whether Ferdinand E. Marcos and his family have the right to travel and liberty of abode, in light of the attendant circumstances in the present case.

HELD: The individual right involved is not the right to travel from the Philippines to other countries or withinthe Philippines. These are what the right to travel would normally connote. Essentially, the right involved isthe right to return to one's country, a totally distinct right under international law, independent from althoughrelated to the right to travel. Thus, the Universal Declaration of Humans Rights and the InternationalCovenant on Civil and Political Rights treat the right to freedom of movement and abode within the territoryof a state, the right to leave a country, and the right to enter one's country as separate and distinct rights. TheDeclaration speaks of the "right to freedom of movement and residence within the borders of each state"separately from the "right to leave any country, including his own, and to return to his country." On the otherhand, the Covenant guarantees the "right to liberty of movement and freedom to choose his residence" and theright to "be free to leave any country, including his own." which rights may be restricted by such laws as "arenecessary to protect national security, public order, public health or morals or the separate rights and freedomsof others." as distinguished from the "right to enter his own country" of which one cannot be "arbitrarilydeprived." It would therefore be inappropriate to construe the limitations to the right to return to one's countryin the same context as those pertaining to the liberty of abode and the right to travel. The right to return toone's country is not among the rights specifically guaranteed in the Bill of Rights, which treats only of theliberty of abode and the right to travel, but the right to return may be considered, as a generally acceptedprinciple of international law and, under our Constitution, is part of the law of the land. However, it is distinctand separate from the right to travel and enjoys a different protection under the International Covenant ofCivil and Political Rights, i.e., against being "arbitrarily deprived" thereof. On the other hand, the Constitution

declares among the guiding principles that "the prime duty of the Government is to serve and protect thepeople" and that "the maintenance of peace and order, the protection of life, liberty, and property, and thepromotion of the general welfare are essential for the enjoyment by all the people of the blessings ofdemocracy." Admittedly, service and protection of the people, the maintenance of peace and order, theprotection of life, liberty and property, and the promotion of the general welfare are essentially ideals to guidegovernmental action. Thus, in the exercise of presidential functions, in drawing a plan of government, and indirecting implementing action for these plans, or from another point of view, in making any decision asPresident of the Republic, the President has to consider these principles, among other things, and adhere tothem. Faced with the problem of whether or not the time is right to allow the Marcoses to return to thePhilippines, the President is, under the Constitution, constrained to consider these basic principles in arrivingat a decision. More than that, having sworn to defend and uphold the Constitution, the President has theobligation under the Constitution to protect the people, promote their welfare and advance the nationalinterest. Since the persons who seek to return to the country are the deposed dictator and his family at whosedoor the travails of the country are laid and from whom billions of dollars believed to be ill-gotten wealth aresought to be recovered, the constitutional guarantees must be adjusted to the requirements of equallyimportant public interests, as such are neither absolute nor inflexible. The President has determined that thedestabilization caused by the return of the Marcoses would wipe away the gains achieved during the past fewyears and lead to total economic collapse. Given what is within our individual and common knowledge of thestate of the economy, the Court cannot argue with that determination.

3.  SILVERIO VS. COURT OF APPEALS [GR 94284, 8 April 1991]

Second Division, Melencio-Herrera (J): 4 concur

Facts: On 14 October 1985, Ricardo C. Silverio was charged with violation of Section 20 (4) of the RevisedSecurities Act in Criminal Case CBU-6304 of the Regional Trial Court of Cebu. In due time, he posted bailfor his provisional liberty. On 26 January 1988, or more than 2 years after the filing of the Information, thePeople of the Philippines filed an Urgent ex parte Motion to cancel the passport of and to issue a hold-departure Order against Silverio on the ground that he had gone abroad several times without the necessaryCourt approval resulting in postponements of the arraignment and scheduled hearings. Overruling opposition,

Page 3: Liberty of Abode and Travel to Freedom of Association

COMPILATION OF CASE DIGEST CONSTI 2: LIBERTY OF ABODE AND TRAVEL AND FREEDOM OF ASSOCIATION ISV Notes the Regional Trial Court, on 4 April 1988, issued an Order directing the Department of Foreign Affairs tocancel Silverio's passport or to deny his application therefor, and the Commission on Immigration to preventSilverio from leaving the country. This order was based primarily on the Trial Court's finding that since thefiling of the Information on 14 October 1985, "the accused has not yet been arraigned because he has neverappeared in Court on the dates scheduled for his arraignment and there is evidence to show that accusedRicardo C. Silverio, Sr. has left the country and has gone abroad without the knowledge and permission ofthis Court." Silverio's Motion for Reconsideration was denied on 28 July 1988. Silverio's Certiorari Petitionbefore the Court of Appeals met a similar fate on 31 January 1990. Hence, the Petition for Review filed on 30July 1990.

Issue: Whether the right to travel can be impaired upon lawful order of the Court, even on grounds other thanthe "interest of national security, public safety or public health."

Held: Article III, Section 6 of the 1987 Constitution should be interpreted to mean that while the liberty oftravel may be impaired even without Court Order, the appropriate executive officers or administrativeauthorities are not armed with arbitrary discretion to impose limitations. They can impose limits only on thebasis of "national security, public safety, or public health" and "as may be provided by law," a limitive phrasewhich did not appear in the 1973 text. Apparently, the phraseology in the 1987 Constitution was a reaction tothe ban on international travel imposed under the previous regime when there was a Travel Processing Center,which issued certificates of eligibility to travel upon application of an interested party. Article III, Section 6 ofthe 1987 Constitution should by no means be construed as delimiting the inherent power of the Courts to useall means necessary to carry their orders into effect in criminal cases pending before them. When by lawjurisdiction is conferred on a Court or judicial officer, all auxiliary writs, process and other means necessaryto carry it into effect may be employed by such Court or officer. Herein, Silverio is facing a criminal charge.He has posted bail but has violated the conditions thereof by failing to appear before the Court when required.Warrants for his arrest have been issued. Those orders and processes would be rendered nugatory if anaccused were to be allowed to leave or to remain, at his pleasure, outside the territorial confines of thecountry. Holding an accused in a criminal case within the reach of the Courts by preventing his departurefrom the Philippines must be considered as a valid restriction on his right to travel so that he may be dealt

with in accordance with law. The offended party in any criminal proceeding is the People of the Philippines. Itis to their best interest that criminal prosecutions should run their course and proceed to finality withoutundue delay, with an accused holding himself amenable at all times to Court Orders and processes.

4.CAUNCA VS SALAZAR

Facts: This is an action for habeas corpus brought by Bartolome Caunca in behalf of his cousin Estelita Flores who was employed by the Far Eastern Employment Bureau, owned by Julia Salazar, respondent herein. An advanced payment has already been given to Estelita by the employment agency, for her to work as a maid. However, Estelita wanted to transfer to another residence, which was disallowed by the employment agency. Further she was detained and her liberty was restrained. The employment agency wanted that the advance payment, which was applied to her transportation expense from the province should be paid by Estelita before she could be allowed to leave.

Issue: Whether or Not an employment agency has the right to restrain and detain a maid without returning the advance payment it gave?

Held: An employment agency, regardless of the amount it may advance to a prospective employee or maid, has absolutely no power to curtail her freedom of movement. The fact that no physical force has been exerted to keep her in the house of the respondent does not make less real the deprivation of her personal freedom of movement, freedom to transfer from one place to another, freedom to choose one’s residence. Freedom may be lost due to external moral compulsion, to founded or groundless fear, to erroneous belief in the existence of an imaginary power of an impostor to cause harm if not blindly obeyed, to any other psychological element that may curtail the mental faculty of choice or the unhampered exercise of the will. If the actual effect of such psychological spell is to place a person at the mercy of another, the victim is entitled to the protection of courts of justice as much as the individual who is illegally deprived of liberty by duress or physical coercion.

5.Villavicencio vs Lukban L-14639

Facts:

Justo Lukban as Manila Mayor together with the police officer, took custody of 170 women at the night of October 25 beyond the latter's consent and knowledge and thereafter were shipped to Davao City where they

Page 4: Liberty of Abode and Travel to Freedom of Association

COMPILATION OF CASE DIGEST CONSTI 2: LIBERTY OF ABODE AND TRAVEL AND FREEDOM OF ASSOCIATION ISV Notes were signed as laborers.A writ of habeas corpus was filed against the mayor on behalf of those women. The court granted the writ, but the mayor was not able to bring any of the women before the court on the stipulated date.

Issue:

Whether or not the act of mayor has a legal basis.

Held:

The supreme court said that the mayor's acts were not legal. His intent of exterminating vice was commendable, but there was no law saying that he could force filipino women to change their domicile from manila to nother place. The women, said the court, although in a sense "lepers of society" were still filipino citizens and such they were entitled to the constitutional enjoyed by all other filipino citizens. The right to freedom of domicile was such a fundamental right that its suppression could considered tantamount to slavery.

The supreme court upheld the right of filipino citizens to freedom of domicile or the Liberty of abode. "Ours is a government of laws and not of men."

6. Angel Lorenzo v. The Director of Health

FACTS:The Petitioner, who suffered from leprosy (Hansen’s disease), was confined in the San Lazaro Hospital inManila in conformity with 1058 of the Administrative Code, which empowered the Director of Health and hisauthorized agents “to cause to be apprehended, and detained, isolated, or confined, all leprous persons in thePhilippine Islands.” The Petitioner brought a petition for habeas corpus, alleging that 1058 was unconstitutional. The Court of First Instance of Manila sustained 1058 and denied the petition for habeas corpus. Thepetitioner appealed this judgment, arguing that the trial court was required to receive evidence to determinewhether or not leprosy was a contagious disease.

HELD:The Court held that Article XV of Chapter 37, 1058 of the Administrative Code was constitutional. Section1058 of the Administrative Code was a legitimate exercise of a police power that extended to the protectionof the public health and leprosy undoubtedly qualified as a serious public health problem. Leprosy wascommonly believed to be an infectious disease and scientific authority supported segregation of lepers as ameans of preventing the spread of the disease. The local legislature had regarded leprosy as a contagiousdisease and had authorized lawful measures to control its spread. Whether or not this determination was

correct, it had been decided according to due process of law, and was not open to the Court to review.

Decision Excerpts“The methods provided for the control of leprosy plainly constitute due process of law. The assumption mustbe that if evidence was required to establish the necessity for the law, that it was before the legislature whenthe act was passed. In the case of a statute purporting the have been enacted in the interest of the publichealth, all questions relating to the determination of matters of fact are for the legislature. If there is probablebasis for sustaining the conclusion reached, its findings are not subject to judicial review. Debatablequestions are for the Legislature to decide. The courts do not sit to resolve the merits of conflicting theories.” “Similarly, the local legislature has regarded leprosy as a contagious disease and has authorized measures tocontrol the dread scourge. To that forum must the petitioner go to reopen the question. We are frank to saythat it would require a much stronger case than the one at bar for us to sanction admitting the testimony ofexpert or other witnesses to show that a law of this character may possibly violate some constitutionalprovision.

FREEDOM OF INFORMATION1. RE: REQUEST FOR COPY OF 2008 STATEMENT OF ASSETS, LIABLITIES AND NETWORTH (SALN) AND PERSONAL DATA SHEET OR CURRICULUM VITAE OF THE JUSTICES OF THE SUPREME COURT AND OFFICERS AND EMPLOYEES OF THE JUDICIARY AM No. 09-8-6- SC, 13 June 2012, EN BANC (Mendoza, J.)[UST Law Review Vol LVII, No. 1, NOVEMBER 2012]

In accordance with section 17, Article IX of the 1987 Philippine Constitution, there is a duty on the part of the members of the judiciary to disclose their SALNs and other personal documents, being a matter of public concern and interest, as long as it is made in a manner provided by law.

The Court received two letters from Rowena C. Paraan and Karol M. Ilagan requesting for copies of the Statement of Assets, Liabilities and Networth (SALN) and Personal Data Sheet (PDS) or the Curriculum Vitae (CV) of its Justices for the year 2008 for the purpose of updating their database of information on government officials. The special committee created by the Supreme Court (SC) to review the request issued a Memorandum recommending the creation of a Committee on Public Disclosure to take over the functions of the Office of the Court Administrator (OCA) with respect to requests for

Page 5: Liberty of Abode and Travel to Freedom of Association

COMPILATION OF CASE DIGEST CONSTI 2: LIBERTY OF ABODE AND TRAVEL AND FREEDOM OF ASSOCIATION ISV Notes copies of SALN and other personal documents of members of the Judiciary. Meanwhile, several and other personal documents of the Justices of the Court, the Court of Appeals (CA) and the Sandigan Bayan (SB) were filed.

ISSUE:Whether or not the Supreme Court Justices are

mandated by law to release their SALN to the public

HELD:Section 7 of Article III of the Constitution is

relevant in the issue of public disclosure of SALN and other documents of public officials, which provides that the right of the people to information on matters of public concern shall be recognized. Access to official records, and to documents, and papers pertaining to official acts, transactions, or decisions, as well as to government research data used as basis for policy development, shall be afforded the citizen, subject to such limitations as may be provided by law.

Emphasizing the import and meaning of the foregoing constitutional provision, the Court in the landmark case of Valmonte v. Belmonte, Jr., elucidated that the right to information goes hand-in-hand with the constitutional policies of full public disclosure and honesty in the public service. It is meant to enhance the widening role of the citizenry in governmental decision making as well as in checking abuse in government. The importance of the said right was pragmatically explicated that the incorporation of this right in the Constitution is a recognition of the fundamental role of free exchange of information in a democracy. There can be no realistic perception by the public of the nation’s problems, nor a meaningful democratic decision-making if they are denied access to information of general interest. Information is needed to enable them members of society to cope with the exigencies of the times. However, restrictions on the access to certain records may be imposed by law.

Thus, while “public concern” like “public interest” eludes exact definition and has been said to embrace a broad spectrum of subjects which the public may want to know, either because such matters directly affect their lives, or simply because such matters naturally arouse the interest of an ordinary citizen, the Constitution itself, under Section 17, Article XI, has classified the information disclosed in the SALN as a matter of public concern and interest. In other words, a “duty to disclose sprang from the “right to know”. Both of constitutional origin, the former is a command while the latter is a permission. Hence, there is a duty on the part of members of the government to disclose their SALN’s to the public in the manner provided by law.

In the case at bar, the Court notes the valid concerns of the other magistrates regarding the possible illicit motives of some individuals in their request for access to such personal information and their

publication. However, custodians of public documents must not concern themselves with motives, reasons and objects of the persons seeking access to the records. The moral or material injury which their misuse might inflict on others is the requestor’s responsibility and lookout. While public officers in the custody or control of public records have the discretion to regulate the manner in which records may be inspected, examined or copied by interested persons, such discretion does not carry with the authority to prohibit access, inspection, examination or copying of the records. After all, public office is a public trust

2. Valmonte Vs Belmonte

FACTS : Petitioners in this special civil action for mandamus with preliminary injunction invoke their right to information and pray that respondent be directed: (a) to furnish petitioners the list of the names of the Batasang Pambansa members belonging to the UNIDO and PDP-Laban who were able to secure clean loans immediately before the February 7 election thru the intercession/marginal note of the then First Lady Imelda Marcos; and/or (b) to furnish petitioners with certified true copies of the documents evidencing their respective loans; and/or (c) to allow petitioners access to the public records for the subject information On June 20, 1986, apparently not having yet received the reply of the Government Service and Insurance System (GSIS) Deputy General Counsel, petitioner Valmonte wrote respondent another letter, saying that for failure to receive a reply, "(W)e are now considering ourselves free to do whatever action necessary within the premises to pursue our desired objective in pursuance of public interest."

ISSUE : WON Valmonte, et. al. are entitled as citizens and taxpayers to inquire upon GSIS records on behest loans given by the former First Lady Imelda Marcos to Batasang Pambansa members belonging to the UNIDO and PDP-Laban political parties.

HELD : Respondent has failed to cite any law granting the GSIS the privilege of confidentiality as regards the documents subject of this petition. His position is apparently based merely on considerations of policy. The judiciary does not settle policy issues. The Court can only declare what the law is, and not what the law should be. Under our system of government, policy issues are within the domain of the political branches of the government, and of the people themselves as the repository of all State power. The concerned borrowers themselves may not succeed if they choose to invoke their right to privacy, considering the public offices they were holding at the time the loans were alleged to have been granted. It cannot be denied that because of the interest they generate and their newsworthiness, public figures, most especially those holding responsible positions in government, enjoy a more limited right to privacy as compared to ordinary individuals, their actions

Page 6: Liberty of Abode and Travel to Freedom of Association

COMPILATION OF CASE DIGEST CONSTI 2: LIBERTY OF ABODE AND TRAVEL AND FREEDOM OF ASSOCIATION ISV Notes being subject to closer public scrutiny The "transactions" used here I suppose is generic and, therefore, it can cover both steps leading to a contract, and already a consummated contract, Considering the intent of the framers of the Constitution which, though not binding upon the Court, are nevertheless persuasive, and considering further that government-owned and controlled corporations, whether performing proprietary or governmental functions are accountable to the people, the Court is convinced that transactions entered into by the GSIS, a government-controlled corporation created by special legislation are within the ambit of the people's right to be informed pursuant to the constitutional policy of transparency in government dealings. Although citizens are afforded the right to information and, pursuant thereto, are entitled to "access to official records," the Constitution does not accord them a right to compel custodians of official records to prepare lists, abstracts, summaries and the like in their desire to acquire information on matters of public concern.

3. Legaspi vs. CSC G.R. No. L-72119, May 29, 1987

Facts: The respondent CSC had denied petitioner Valentin Legaspi’s request for information on the civil service eligibilities of Julian Sibonghanoy and Mariano Agas who were employed as sanitarians in the Health Department of Cebu City. Sibonghanoy and Agas had allegedly represented themselves as civil service eligibles who passed the civil service examinations for sanitarians.

Claiming that his right to be informed of the eligibilities of Sibonghanoy and Agas is guaranteed by the Constitution, and that he has no other plain, speedy and adequate remedy to acquire the information, petitioner prays for the issuance of the extraordinary writ of mandamus to compel the respondent CSC to disclose said information.

The respondent CSC takes issue on the personality of the petitioner to bring the suit. It is asserted that the petition is bereft of any allegation of Legaspi’s actual interest in the civil service eligibilities of Sibonghanoy and Agas.

Issue: Whether or not the petitioner has legal standing to bring the suit

Held: The petitioner has firmly anchored his case upon the right of the people to information on matters of public concern, which, by its very nature, is a public right. It has been held in the case of Tanada vs. Tuvera, 136 SCRA 27, that when the question is one of public right and the object of the mandamus is to procure the enforcement of a public duty, the people are regarded as the real party in interest, and the person at whose instigation the

proceedings are instituted need not show that he has any legal or special interest in the result, it being sufficient to show that he is a citizen and as such interested in the execution of the laws.

It becomes apparent that when a mandamus proceeding involves the assertion of a public right, the requirement of personal interest is satisfied by the mere fact that the petitioner is a citizen, and therefore, part of the general public which possesses the right.

The petitioner, being a citizen who as such, is clothed with personality to seek redress for the alleged obstruction of the exercise of the public right. 

4.Bantay vs COMELEC

Facts: There were two consolidated petitions forcertiorari and mandamus to nullify and set aside certain issuances of the Commission on Elections (Comelec) respecting party-list groups which have manifested their intention to participate in the party-list elections on May 14, 2007.In the first petition, docketed as G.R. No. 177271, petitioners Bantay Republic Act (BA-RA 7941, for short) and the Urban Poor for Legal Reforms (UP-LR, for short) assail the various Comelec resolutions accrediting private respondents Biyaheng Pinoy et al., to participate in the forthcoming party-list elections on May 14, 2007 without simultaneously determining whether or not their respective nominees possess the requisite qualifications defined in Republic Act (R.A.) No. 7941, or the "Party-List System Act" and belong to the marginalized and underrepresented sector each seeks to represent. In the second, docketed as G.R. No. 177314, petitioners Loreta Ann P. Rosales, Kilosbayan Foundation and Bantay Katarungan Foundation impugn Comelec Resolution 07-0724 dated April 3, 2007 effectively denying their request for the release or disclosure of the names of the nominees of the fourteen (14)accredited participating party-list groups mentioned in petitioner Rosales’ previous letter -request While both petitions commonly seek to compel the Comelec to disclose or publish the names of the nominees of the various party-list groups named in the petitions, BA-RA 7941 and UP-LR have the additional prayers that the 33 private respondents named therein be “declare[d] as unqualified to participate in the party-list elections and that the Comelec be enjoined from allowing respondent groups from participating in the elections.

Issues: 1. Can the Court cancel the accreditation accorded by the COMELEC to the respondent party-list groups named in their petition on the ground that these groups and their respective nominees do not appear to be qualified?2. Whether respondent COMELEC, by refusing to reveal the names of the nominees of the various party-list groups, has violated the right to information and free

Page 7: Liberty of Abode and Travel to Freedom of Association

COMPILATION OF CASE DIGEST CONSTI 2: LIBERTY OF ABODE AND TRAVEL AND FREEDOM OF ASSOCIATION ISV Notes access to documents as guaranteed by the Constitution; and3. Whether respondent COMELEC is mandated by the Constitution to disclose to the public the names of said nominees.

Ruling:1. The Court is unable to grant the desired plea of petitioners BA-RA 7941 and UP-LR for cancellation of accreditation on the grounds thus advanced in their petition. The exercise would require the Court to make a factual determination, a matter which is outside the office of judicial review by way of special civil action for certiorari. In certiorari proceedings, the Court is not called upon to decide factual issues and the case must be decided on the undisputed facts on record. The sole function of a writ of certiorari is to address issues of want of jurisdiction or grave abuse of discretion and does not include a review of the tribunal’s evaluation of the evidence. (note that nowhere in R.A. No. 7941 is there a requirement that the qualification of a party-list nominee be determined simultaneously with the accreditation of an organization. )

2. COMELEC’s basis of its refusal to disclose the names of the nominees of subject party-list groups,Section 7 of R.A. 7941,which last sentence reads: “[T]he names of the party-list nominees shall not beshown on the certified list” is certainly not a justifying card for the Comelec to deny the requesteddisclosure. There is absolutely nothing in R.A. No. 7941 that prohibits the Comelec from disclosing or even publishing through mediums other than the “Certified List” of the names.

Assayed against the non-disclosure stance of the Comelec and the given rationale therefor is the right to information enshrined in the self-executory 15 Section 7, Article III of the Constitution, viz

:Sec.7. The right of the people to information on matters of public concern shall be recognized. Access to official records, and to documents, and papers pertaining to official acts, transactions, or decisions, as well to government research data used as basis for policy development, shall be afforded the citizen, subject to such limitations as may be provided by law. Complementing and going hand in hand with the right to information is another constitutional provision enunciating the policy of full disclosure and transparency in Government. We refer to Section 28, Article II of the Constitution reading: Sec. 28. Subject to reasonable conditions prescribed by law, the State adopts and implements a policy of full public disclosure of all its transactions involving public interest. It has been repeatedly said in various contexts that the people have the right to elect their representatives on the basis of an informed judgment. Hence the need for voters to be informed about matters that have a bearing on their

choice While the vote cast in a party-list elections is a vote for a party, such vote, in the end, would be a vote for its nominees, who, in appropriate cases, would eventually sit in the House of Representatives. The Court frowns upon any interpretation of the law or rules that would hinder in anyway the free and intelligent casting of the votes in an election.

3.COMELEC has a constitutional duty to disclose and release the names of the nominees of the party-list groups named in the herein petitions. The right to information is a public right where the real parties in interest are the public, or the citizens to be precise, but like all constitutional guarantees, however, the right to information and its companion right of access to official records are not absolute. The people’s right to know is limited to “matters of public concern” and is further subject to such limitation as may be provided by law. But no national security or like concerns is involved in the disclosure of the names of the nominees of the party-list groups in question. Doubtless, the Comelec committed grave abuse of discretion in refusing the legitimate demands of the petitioners for a list of the nominees of the party-list groups subject of their respective petitions. The 1st petition is partly DENIED insofar as it seeks to nullify the accreditation of the respondents named therein. However, insofar as it seeks to compel the Comelec to disclose or publish the names of the nominees of party-list groups, sectors or organizations accredited to participate in the May 14, 2007elections, the 2 petitions are GRANTED. Accordingly, the Comelec is hereby ORDERED to immediately disclose and release the names of the nominees of the party-list groups.

5. Echegaray vs Secretary of JusticeFacts:The Supreme Court affirmed the conviction of petitioner Leo Echegaray y Pilo for the crime of rape of the 10 year-old daughter of his common-law spouse. The supreme penalty of death was to be imposed upon him. He then filed motion for recon and a supplemental motion for recon raising constitutionality of Republic Act No. 7659 and the death penalty for rape. Both were denied. Consequently, Congress changed the mode of execution of the death penalty from electrocution to lethal injection, and passed Republic Act No. 8177, designating death by lethal injection. Echegaray filed a Petition for prohibition from carrying out the lethal injection against him under the grounds that it constituted 1. cruel, degrading, or unusual punishment, 2. Being violative of due process, 3. a violation of the Philippines’ obligations under international covenants, 4. an undue delegation of legislative power by Congress, an unlawful exercise by respondent Secretary of the power to legislate, and an unlawful delegation of

Page 8: Liberty of Abode and Travel to Freedom of Association

COMPILATION OF CASE DIGEST CONSTI 2: LIBERTY OF ABODE AND TRAVEL AND FREEDOM OF ASSOCIATION ISV Notes delegated powers by the Secretary of Justice. In his motion to amend, the petitioner added equal protection as a ground.

The Solicitor General stated that the Supreme Court has already upheld the constitutionality of the Death Penalty Law, and has declared that the death penalty is not cruel, unjust, excessive or unusual punishment; execution by lethal injection, as authorized under R.A. No. 8177 and the questioned rules, is constitutional, lethal injection being the most modern, more humane, more economical, safer and easier to apply (than electrocution or the gas chamber); in addition to that, the International Covenant on Civil and Political Rights does not expressly or impliedly prohibit the imposition of the death penalty.

Issues: 1. Is the lethal injection a cruel, degrading or inhuman punishment? 2. Is it a violation of our international treaty obligations? 3. Is it discriminatory (pertaining to sec 17)?

Held: 1. No 2. Yes 3rd. Petition denied.

1. Petitioner contends that death by lethal injection constitutes cruel, degrading and inhuman punishment because (1) R.A. No. 8177 fails to provide for the drugs to be used in carrying out lethal injection, the dosage for each drug to be administered, and the procedure in administering said drug/s into the accused; (2) its implementing rules are uncertain as to the date of the execution, time of notification, the court which will fix the date of execution, which uncertainties cause the greatest pain and suffering for the convict; and (3) the possibility of mistakes in administering the drugs renders lethal injection inherently cruel. It is well-settled in jurisprudence that the death penalty per se is not a cruel, degrading or inhuman punishment. In Harden v. Director of Prisons- “punishments are cruel when they involve torture or a lingering death; but the punishment of death is not cruel, within the meaning of that word as used in the constitution. It implies there something inhuman and barbarous, something more than the mere extinguishment of life.” Would the lack in particularity then as to the details involved in the execution by lethal injection render said law “cruel, degrading or inhuman”? The Court believes not. Petitioner contends that Sec. 16

of R.A. No. 8177 is uncertain as to which “court” will fix the time and date of execution, and the date of execution and time of notification of the death convict. As petitioner already knows, the “court” which designates the date of execution is the trial court which convicted the accused. The procedure is that the “judgment is entered fifteen (15) days after its promulgation, and 10 days thereafter, the records are remanded to the court below including a certified copy of the judgment for execution. Neither is there any uncertainty as to the date of execution nor the time of notification. As to the date of execution, Section 15 of the implementing rules must be read in conjunction with the last sentence of Section 1 of R.A. No. 8177 which provides that the death sentence shall be carried out “not earlier than one (1) year nor later then eighteen (18) months from the time the judgment imposing the death penalty became final and executory, without prejudice to the exercise by the President of his executive clemency powers at all times.” Hence, the death convict is in effect assured of eighteen (18) months from the time the judgment imposing the death penalty became final and executor wherein he can seek executive clemency and attend to all his temporal and spiritual affairs. Petitioner also contends that the infliction of “wanton pain” in case of possible complications in the intravenous injection that respondent Director is an untrained and untested person insofar as the choice and administration of lethal injection is concerned, renders lethal injection a cruel, degrading and inhuman punishment. This is unsubstantiated. First. Petitioner has neither alleged nor presented evidence that lethal injection required the expertise only of phlebotomists and not trained personnel and that the drugs to be administered are unsafe or ineffective. Petitioner simply cites situations in the United States wherein execution by lethal injection allegedly resulted in prolonged and agonizing death for the convict, without any other evidence whatsoever. Second. Petitioner overlooked Section 1, third paragraph of R.A. No. 8177 which requires that all personnel involved in the execution proceedings should be trained prior to the performance of such task. We must presume that the public officials entrusted with the implementation of the death penalty will carefully avoid inflicting cruel punishment. Third. Any infliction of pain in lethal injection is merely incidental in carrying out the execution of death penalty and does not fall within the constitutional proscription against cruel, degrading and inhuman punishment. “In a limited sense, anything is cruel which is calculated to give pain or distress, and since punishment imports pain or suffering to the convict, it may be said that all punishments are cruel. But of course the Constitution does not mean that crime, for this reason, is to go unpunished.” The cruelty

Page 9: Liberty of Abode and Travel to Freedom of Association

COMPILATION OF CASE DIGEST CONSTI 2: LIBERTY OF ABODE AND TRAVEL AND FREEDOM OF ASSOCIATION ISV Notes against which the Constitution protects a convicted man is cruelty inherent in the method of punishment, not the necessary suffering involved in any method employed to extinguish life humanely.

2. Violation of international treaties? In countries which have not abolished the death penalty, sentence of death may be imposed only for the most serious crimes in accordance with the law in force at the time of the commission of the crime and not contrary to the provisions of the present Covenant and to the Convention on the Prevention and Punishment of the Crime of Genocide. This penalty can only be carried out pursuant to a final judgment rendered by a competent court. The punishment was subject to the limitation that it be imposed for the “most serious crimes”. Included with the declaration was the Second Optional Protocol to the International Covenant on Civil and Political Rights, Aiming at the Abolition of the Death Penalty was adopted by the General Assembly on December 15, 1989. The Philippines neither signed nor ratified said document.

3. Petitioner contends that Section 17 of the Implementing Rules is unconstitutional for being discriminatory. “SEC. 17. SUSPENSION OF THE EXECUTION OF THE DEATH SENTENCE. Execution by lethal injection shall not be inflicted upon a woman within the three years next following the date of the sentence or while she is pregnant, nor upon any person over seventy (70) years of age. In this latter case, the death penalty shall be commuted to the penalty of reclusion perpetua with the accessory penalties provided in Article 40 of the Revised Penal Code.” Petitioner contends that Section 17 amends the instances when lethal injection may be suspended, without an express amendment of Article 83 of the Revised Penal Code, as amended by section 25 of R.A. No. 7659, stating that the death sentence shall not be inflicted upon a woman while she is pregnant or within one (1) year after delivery, nor upon any person over seventy years of age. While Article 83 of the Revised Penal Code, as amended by Section 25 of Republic Act No. 7659, suspends the implementation of the death penalty while a woman is pregnant or within one (1) year after delivery, Section 17 of the implementing rules omits the one (1) year period following delivery as an instance when the death sentence is suspended, and adds a ground for suspension of sentence no longer found under Article 83 of the Revised Penal Code as amended, which is the

three-year reprieve after a woman is sentenced. This addition is, in petitioner’s view, tantamount to a gender-based discrimination. Being an implementing rule, Section 17 must not override, but instead remain consistent and in harmony with the law it seeks to implement.

FREEDOM OF ASSOCIATION

1. SSS Employees Assoc. v CA

Facts: On June 11, 1987, the SSS filed with the Regional Trial Court of Quezon City a complaint for damages with a prayer for a writ of preliminary injunction against petitioners, alleging that on June 9, 1987, the officers and members of SSSEA staged an illegal strike and baricaded the entrances to the SSS Building, preventing non-striking employees from reporting for work and SSS members from transacting business with the SSS; that the strike was reported to the Public Sector Labor - Management Council, which ordered the strikers to return to work; that the strikers refused to return to work; and that the SSS suffered damages as a result of the strike. The complaint prayed that a writ of preliminary injunction be issued to enjoin the strike and that the strikers be ordered to return to work; that the defendants (petitioners herein) be ordered to pay damages; and that the strike be declared illegal.

It appears that the SSSEA went on strike after the SSS failed to act on the union's demands, which included: implementation of the provisions of the old SSS-SSSEA collective bargaining agreement (CBA) on check-off of union dues; payment of accrued overtime pay, night differential pay and holiday pay; conversion of temporary or contractual employees with six (6) months or more of service into regular and permanent employees and their entitlement to the same salaries, allowances and benefits given to other regular employees of the SSS; and payment of the children's allowance of P30.00, and after the SSS deducted certain amounts from the salaries of the employees and allegedly committed acts of discrimination and unfair labor practices.

Page 10: Liberty of Abode and Travel to Freedom of Association

COMPILATION OF CASE DIGEST CONSTI 2: LIBERTY OF ABODE AND TRAVEL AND FREEDOM OF ASSOCIATION ISV Notes Issue: Whether or not employees of the Social Security System (SSS) have the right to strike.

Held: The 1987 Constitution, in the Article on Social Justice and Human Rights, provides that the State "shall guarantee the rights of all workers to self-organization, collective bargaining and negotiations, and peaceful concerted activities, including the right to strike in accordance with law" [Art. XIII, Sec. 31].

Resort to the intent of the framers of the organic law becomes helpful in understanding the meaning of these provisions. A reading of the proceedings of the Constitutional Commission that drafted the 1987 Constitution would show that in recognizing the right of government employees to organize, the commissioners intended to limit the right to the formation of unions or associations only, without including the right to strike.

Considering that under the 1987 Constitution "the civil service embraces all branches, subdivisions, instrumentalities, and agencies of the Government, including government-owned or controlled corporations with original charters" [Art. IX(B), Sec. .2(l) see also Sec. 1 of E.O. No. 180 where the employees in the civil service are denominated as "government employees"] and that the SSS is one such government-controlled corporation with an original charter, having been created under R.A. No. 1161, its employees are part of the civil service [NASECO v. NLRC, G.R. Nos. 69870 & 70295, November 24,1988] and are covered by the Civil Service Commission's memorandum prohibiting strikes. This being the case, the strike staged by the employees of the SSS was illegal.

2. Occena v. COMELEC Facts: The challenge in these two prohibition proceedings against the validity of three Batasang Pambansa Resolutions proposing constitutional  amendments goes further than merely assailing their alleged constitutional infirmity. The rather unorthodox aspect of these petitions is the assertion that the 1973 Constitution is not the fundamental law.

The three Resolutions were:

1) Resolution No. 1 proposing an amendment allowing a natural-born citizen of the Philippines naturalized in a foreign country to own a limited area of land for residential purposes

2) Resolution No. 2 dealing with the Presidency, the Prime Minister and the Cabinet, and the National Assembly; and

3) Resolution No. 3 on the amendment to the Article on the Commission on Elections. The three resolutions were approved by the Interim

Batasang Pambansa sitting as a constituent assembly on February 5 and 27, 1981which the date of plebiscite has been set on April 7, 1981. It is thus within the 90-day period provided by the Constitution.

Issues:(1) Whether or not the 1973 Constitution is a fundamental law.(2) Whether or not the Interim BatasangPambansa has the power to propose amendments.(3) Whether or not the three-fourth votes is necessary to propose amendments as wellas the standard for proper submission.(4) Whether or not the three BatasangPambansa Resolutions proposing constitutionalamendments are valid.

Held: Yes, the Interim Batasang Pambansa has the power and privilege to propose amendments. On January 17, 1973, the present Constitution came into force and effect. With such a pronouncement by the Supreme Court and with the recognition of the cardinal postulate that what the Supreme Court says is not only entitled to respect but must also be obeyed, a factorfor instability was removed. Thereafter, as a matter of law, all doubts were resolved. The 1973Constitution is the fundamental law. The existence of this power is indubitable as the applicableprovision in the 1976 Amendments is quite explicit.The Interim Batasang Pambansa, sitting as a constituent body, can propose amendments. In that capacity, only a majority vote is needed. It would be an indefensible proposition to assert that the three-fourth votes required when it sits as a legislative body applies as well when it has been convened as the agency through which amendments could be proposed. That is not a requirement as far as a constitutional convention is concerned. It is not a requirement either when, as in this case, the Interim Batasang Pambansa exercises its constituent power to propose amendments. Resolution No. 1 proposing an amendment allowing a natural-born citizen of the Philippines naturalized in a foreign country to own a limited area of land for residential purposes was approved by the vote of 122 to 5; Resolution No. 2 dealing with the Presidency, the Prime Minister and the Cabinet, and the National Assembly by a vote of 147 to 5 with 1 abstention; and Resolution No. 3 on the amendment to the Article on the Commission on Elections by a vote of 148 to 2 with 1 abstention The three resolutions were approved by the InterimBatasang Pambansa sitting as a constituent assembly on February 5 and 27, 1981,thus making them valid.

Page 11: Liberty of Abode and Travel to Freedom of Association

COMPILATION OF CASE DIGEST CONSTI 2: LIBERTY OF ABODE AND TRAVEL AND FREEDOM OF ASSOCIATION ISV Notes

3. In Re EdillonFacts: This is an administrative case against Edillon who refuses to pay his IBP membership dues assailing the provisions of the Rule of Court 139-A and the provisions of par. 2, Section 24, Article III, of the IBP By-Laws pertaining to the organization of IBP, payment of membership fee and suspension for failure to pay the same. He contends that the stated provisions constitute an invasion of his constitutional rights of being compelled to be a member of the IBP in order to practice his profession and thus deprives his rights to liberty and property and thereby null and void.

Issue: Whether or not it assailed provisions constitutes a deprivation of liberty and property of the respondent.

Held: The court held that the IBP is a State-organized Bar as distinguished from bar associations that are organized by individual lawyers themselves, membership of which is voluntary. The IBP however is an official national body of which all lawyers must be a member and are subjected to the rules prescribed for the governance of the Bar which includes payment of reasonable annual fee for the purpose of carrying out its objectives and implementation of regulations in the practice of law. The provisions assailed does not infringe the constitutional rights of the respondent as it is a valid exercise of police power necessary to perpetuate its existence with regulatory measures to implement. The name of Edillon was stricken out from the rolls of attorney for being a delinquent member of the bar.

4. Sta. Clara Homeowners Association v. GastonFacts: Spouses Victor Ma. Gaston and Lydia M. Gaston were residents of San Jose Avenue, Sta. Clara Subdivision, Mandalagan, Bacolod City. They purchased their lots in the said subdivision sometime in 1974, and at the time of purchase, there was no mention or requirement of membership in any homeowners' association. From that time on, they have remained non-members of SCHA. They also stated that an arrangement was made wherein homeowners who were non-members of the association were issued "non-member" gatepass stickers for their vehicles for identification by the security guards manning the subdivision's entrances and exits. This arrangement remained undisturbed until sometime in the middle of March 1998, when SCHA disseminated a board resolution which decreed that only its members in good standing were to be issued stickers for use in their vehicles. Thereafter, on three separate incidents, Victor M. Gaston, the son of the spouses Gaston who lives with them, was required by the guards on duty employed by SCHA to show his driver's license as a prerequisite to his entrance to the subdivision and to his residence therein despite their knowing him personally and the exact location of his residence.

On 29 March 1998, Victor Ma. Gaston was himself prevented from entering the subdivision and proceeding to his residential abode when security guards Roger Capillo and a "John Doe" lowered the steel bar of the KAMETAL gate of the subdivision and demanded from him his driver's license for identification. On 1 April 1998, Spouses Victor Ma. Gaston and Lydia M. Gaston filed a complaint for damages with preliminary injunction/preliminary mandatory injunction and temporary restraining order before the Regional Trial Court in Negros Occidental at Bacolod City against Santa Clara Homeowners Association (SCHA) thru its Board of Directors, namely: Arneil Chua, Luis Sarrosa, Jocelyn Garcia, Ma. Milagros Vargas, Lorenzo Lacson, Ernesto Piccio, Dindo Ilagan, Danilo Gamboa, Jr., Rizza de la Rama and Security Guard Capillo and 'John Doe', and Santa Clara Estate, Incorporated (Civil Case 98-10217, RTC-Branch 49, Bacolod City); alleging that the acts of SCHA, et al., done in the presence of other subdivision owners had caused the spouses Gaston to suffer moral damage. On 8 April 1998, SCHA, et al. filed a motion to dismiss arguing that the trial court had no jurisdiction over the case as it involved an intra-corporate dispute between SCHA and its members pursuant to Republic Act 580, as amended by Executive Orders 535 and 90, much less, to declare as null and void the subject resolution of the board of directors of SCHA, the proper forum being the Home insurance and Guaranty Corporation (HIGC). To support their claim of intra-corporate controversy, SCHA, et al. stated that the Articles of Incorporation of SCHA, which was duly approved by the Securities and Exchange Commission (SEC) on 4 October 1973, provides "that the association shall be a non-stock corporation with all homeowners of Sta. Clara constituting its membership"; and that its by-laws contains a provision that "all real estate owners in Sta. Clara Subdivision automatically become members of the association"; among others. On 6 July 1998, the lower court resolved to deny SCHA et al.'s motion to dismiss, finding that there existed no intra-corporate controversy since the Spouses Gaston alleged that they had never joined the association.

On 18 July 1998, SCHA, et al. submitted a Motion for Reconsideration, adding lack of cause of action as ground for the dismissal of the case. On 17 August 1998, the trial court denied the said motion without however ruling on the additional ground of lack of cause of action. On 18 August 1998, SCHA, et al. filed a motion to resolve its motion to dismiss on ground of lack of cause of action. On 8 September 1998, the trial court issued an order denying the motion. On 24 September 1998, SCHA. et al. elevated the matter to the Court of Appeals via a Petition for Certiorari. On 31 August 1999, the Court of Appeals dismissed the Petition and ruled that the RTC had jurisdiction over the dispute. The appellate court likewise denied SCHA, et al.'s motion for reconsideration in a resolution dated 11 February 2000. SCHA, et al. filed the petition for review.

Page 12: Liberty of Abode and Travel to Freedom of Association

COMPILATION OF CASE DIGEST CONSTI 2: LIBERTY OF ABODE AND TRAVEL AND FREEDOM OF ASSOCIATION ISV Notes Issue: Whether the Spouses Gaston are members of the SCHA.

Held: The constitutionally guaranteed freedom of association includes the freedom not to associate. The right to choose with whom one will associate oneself is the very foundation and essence of that partnership. Further, the Spouses Gaston cannot be compelled to become members of the SCHA by the simple expedient of including them in its Articles of Incorporation and By-laws without their express or implied consent. True, it may be to the mutual advantage of lot owners in a subdivision to band themselves together to promote their common welfare, but that is possible only if the owners voluntarily agree, directly or indirectly, to become members of the association. True also, memberships in homeowners' associations may be acquired in various ways — often through deeds of sale, Torrens certificates or other forms of evidence of property ownership. Herein, however, other than the said Articles of Incorporation and By-laws, there is no showing that the Spouses Gaston have agreed to be SCHA members. The approval by the SEC of the said documents is not an operative act which bestows membership on the Spouses Gaston because the right to associate partakes of the nature of freedom of contract which can be exercised by and between the homeowners amongst themselves, the homeowners' association and a homeowner, and the subdivision owner and a homeowner/lot buyer. Clearly, there is no privity of contract exists between SCHA and Spouses Gaston. When the Spouses Gaston purchased their property in 1974 and obtained Transfer Certificates of Titles T-126542 and T-127462 for Lots 11 and 12 of Block 37 along San Jose Avenue in Sta. Clara Subdivision, there was no annotation showing their automatic membership in the SCHA. Furthermore, the records are bereft of any evidence that would indicate that the Spouses Gaston intended to become members of the SCHA. Prior to the implementation of the aforesaid Resolution, they and the other homeowners who were not members of the association were issued non-member gate pass stickers for their vehicles; a fact not disputed by SCHA. Thus, the SCHA recognized that there were subdivision landowners who were not members thereof, notwithstanding the provisions of its Articles of Incorporation and By-laws.

5. Alliance of Concerned Government Workers v. Minister of Labor and Employment

GUTIERREZ, JR., J.:

Are the branches, agencies, subdivisions, and instrumentalities of the Government, including government owned or controlled corporations included among the 4 "employers"" under Presidential Decree No. 851 which are required to pay an their employees receiving a basic salary of not more than P1,000.00 a

month, a thirteenth (13th) month pay not later than December 24 of every year?

Petitioner Alliance of Government Workers (AGW) is a registered labor federation while the other petitioners are its affiliate unions with members from among the employees of the following offices, schools, or government owned or controlled corporations:

1. Philippine National Bank (PNB) Escolta Street, Manila

2. Metropolitan Waterworks and Sewerage System (MWSS) Katipunan Road, Balara, Quezon City

3. Government Service Insurance System (GSIS) Arroceros Street, Manila

4. Social Security System (SSS) East Avenue, Quezon City

5. Philippine Virginia Tobacco Administration (PVTA) Consolacion Building, Cubao, Quezon City

6. Philippine Normal College (PNC) Ayala Boulevard, Manila

7. Polytechnic University of the Philippines (PUP) Hippodromo Street, Sta. Mesa, Manila

On February 28, 1983, the Philippine Government Employees Association (PGEA) filed a motion to come in as an additional petitioner.

Presidential Decree No. 851 provides in its entirety:

WHEREAS, it is necessary to further protect the level of real f wages from the ravage of world-wide inflation;

WHEREAS, there has been no increase case in the legal minimum wage rates since 1970;

WHEREAS, the Christmas season is an opportune time for society to show its concern for the plight of the working masses so they may properly celebrate Christmas and New Year.

NOW, THEREFORE, I, FERDINAND E. MARCOS, by virtue of the powers vested in me by the Constitution do hereby decree as follows:

Page 13: Liberty of Abode and Travel to Freedom of Association

COMPILATION OF CASE DIGEST CONSTI 2: LIBERTY OF ABODE AND TRAVEL AND FREEDOM OF ASSOCIATION ISV Notes SECTION 1. All employers are hereby required to pay all their employees receiving a basic salary of not more than Pl,000 a month, regardless of the nature of their employment, a 13th-month pay not later than December 24 of every year.

SECTION 2. Employers already paying their employees a 13th-month pay or its equivalent are not covered by this Decree.

SECTION 3. This Decree shall take effect immediately. Done in the City of Manila, this 16th day of December 1975.

According to the petitioners, P.D. No. 851 requires all employers to pay the 13th-month pay to their employees with one sole exception found in Section 2 which states that "(E)mployers already paying their employees a 13th month pay or its equivalent are not covered by this Decree. " The petitioners contend that Section 3 of the Rules and Regulations Implementing Presidential Decree No. 851 included other types of employers not exempted by the decree. They state that nowhere in the decree is the secretary, now Minister of Labor and Employment, authorized to exempt other types of employers from the requirement.

Section 3 of the Rules and Regulations Implementing Presidential Decree No. 851 provides:

Section 3. Employers covered — The Decree shall apply to all employers except to:

a) Distressed employers, such as (1) those which are currently incurring substantial losses or 112) in the case of non-profit institutions and organizations, where their income, whether from donations, contributions, grants and other earnings from any source, has consistently declined by more than forty (40%) per cent of their normal income for the last two (2) )years, subject to the provision of Section 7 of this issuance.

b) The Government and any of its political subdivisions, including government-owned and controlled corporations, except)t those corporation, operating essentially as private, ,subsidiaries of the government;

c) Employers already paying their employees 13th-month pay or more in a calendar year or its equivalent at the of this issuance;

d) Employers of household helpers and persons in the personal service of another in relation to such workers: and

e) Employers of those who are paid on purely commission, boundary, or task basis and those who are paid a fixed for performing a specific work, irrespective of the time consumed in the performance thereof, except where the workers are paid an piece- rate basis in which case the employer shall be covered by this issuance :insofar ab such workers are concerned ...

The petitioners assail this rule as ultra vires and void. Citing Philippine Apparel Workers'Union v. NIRC et al., (106 SCRA 444); Teoxon v. Members of the Board of' Administators (33 SCRA 585); Santos u. Hon. Estenzo et al., (109 Phil. 419); Hilado u. Collector of Internal Revenue (100 Phil. 288), and Olsen & Co. Inc. v. Aldanese and Trinidad (43 Phil. 259), the petitioners argue that regulations adopted under legislative authority must be in harmony with the provisions of the law and for the sole purpose of carrying into effect its general provisions. They state that a legislative act cannot be amended by a rule and an administrative officer cannot change the law. Section 3 is challenged as a substantial modification by rule of a Presidential Decree and an unlawful exercise of legislative power.

Our initial reaction was to deny due course to the petition in a minute resolution, however, considering the important issues propounded and the fact, that constitutional principles are involved, we have now decided to give due course to the petition, to consider the various comments as answers and to resolve the questions raised through a full length decision in the exercise of this Court's symbolic function as an aspect of the power of judicial review.

At the outset, the petitioners are faced with a procedural barrier. The petition is one for declaratory relief, an action not embraced within the original jurisdiction of the Supreme Court. (Remotigue v. Osmena,, Jr., 21 SCRA 837; Rural Bank of Olongapo v. Commission of Land Registration, 102 SCRA 794; De la Llana v. Alba, 112 SCRA 294). There is no statutory or jurisprudential basis for the petitioners' statement that the Supreme Court has original and exclusive jurisdiction over declaratory relief suits where only questions of law are involved. Jurisdiction is conferred by law. The petitioners have not pointed to any provision of the Constitution or statute which sustains their sweeping assertion. On this ground alone, the petition could have been dismissed outright.

Following similar action taken in Nacionalista Party v. Angelo Bautista (85 Phil. 101) and Aquino v. Commission on Elections (62 SCRA 275) we have, however, decided to treat the petition as one for

Page 14: Liberty of Abode and Travel to Freedom of Association

COMPILATION OF CASE DIGEST CONSTI 2: LIBERTY OF ABODE AND TRAVEL AND FREEDOM OF ASSOCIATION ISV Notes mandamus. The petition has far reaching implications and raises questions that should be resolved. Have the respondents unlawfully excluded the petitioners from the use and enjoyment of rights to which they are entitled under the law?

An analysis of the "whereases" of P.D. No. 851 shows that the President had in mind only workers in private employment when he issued the decree. There was no intention to cover persons working in the government service. The decree states:

xxx xxx xxx

WHEREAS, there has been no increase in the legal minimum wage rates since 1970;

xxx xxx xxx

As pointed out by the Solicitor General in his comment for the Minister of Labor and Employment, the Social Security System the Philippine Normal College, and Polytechnic University, the contention that govermment owned and controlled corporations and state colleges and universities are covered by the term "all employers" is belied by the nature of the 13- month pay and the intent behind the decree.

The Solicitor General states:

"Presidential Decree No. 851 is a labor standard law which requires covered employers to pay their employees receiving not more than P1,000.00 a month an additional thirteenth-month pay. Its purpose is to increase the real wage of the worker (Marcopper Mining Corp. v. Ople, 105 SCRA 75; and National Federation of Sugar Workers v. Ovejera, G.R. No. 59743, May 31, 1982) as explained in the'whereas'clause which read:

WHEREAS, it is necessary to further protect the level of real wages from the ravage of world-wide inflation;

WHEREAS, there has been no increase in the legal minimum wage rates since 1970; 11

WHEREAS, the Christmas season is an opportune time for society to show its concern for the plight of the working masses so they may celebrate the

Christmas and New Year.

xxx xxx xxx

What the P.D. No. 851 intended to cover, as explained in the prefatory statement of the Decree, are only those in the private sector whose real wages require protection from world-wide inflation. This is emphasized by the "whereas" clause which states that 'there has been no increase in the legal minimum wage rates since 1970'. This could only refer to the private sector, and not to those in the government service because at the time of the enactment of Presidential Decree No. 851 in 1975, only the employees in the private sector had not been given any increase in their minimum wage. The employees in the government service had already been granted in 1974 a ten percent across-the-board increase on their salaries as stated in P.D. No. 525, Section 4.

Moreover, where employees in the government service were to benefit from labor standard laws, their coverage is explicitly stated in the statute or presidential enactment. This is evident in (a) Presidential Decree No. 390, Sec. 1 which granted emergency cost of living allowance to employees in the national government; (b) Republic Act No. 6111, Sec. 10 on medicare benefits; (c) Presidential Decree No -442, Title II, Article 97 on the applicable minimum wage rates; (d) Presidential Decree No. 442, Title 11, Article 167 (g) on workmen's compensation; (e) Presidential Decree No. 1123 which provides for increases in emergency allowance to employees in the private sector and in salary to government employees in Section 2 thereof; and (f) Executive Order No. 752 granting government employees a year-end bonus equivalent to one week's pay. Thus, had the intention been to include government employees under the coverage of Presidential Decree No. 851, said Decree should have expressly so provided and there should have been accompanying yearly appropriation measures to implement the same. That no such express provision was provided and no accompanying appropriation measure to was passed clearly show the intent to exclude government

Page 15: Liberty of Abode and Travel to Freedom of Association

COMPILATION OF CASE DIGEST CONSTI 2: LIBERTY OF ABODE AND TRAVEL AND FREEDOM OF ASSOCIATION ISV Notes employees from the coverage of P. D. No. 85 1.

We agree.

It is an old rule of statutory construction that restrictive statutes and acts which impose burdens on the public treasury or which diminish rights and interests, no matter how broad their terms do not embrace the Sovereign, unless the Sovereign is specifically mentioned. (See Dollar Savings Bank v. United States, 19 Wall (U.S.) 227; United States v. United Mine Workers of America, 330 U.S. 265). The Republic of the Philippines, as sovereign, cannot be covered by a general term like "employer" unless the language used in the law is clear and specific to that effect.

The issue raised in this petition, however, is more basic and fundamental than a mere ascertainment of intent or a construction of statutory provisions. It is concerned with a revisiting of the traditional classification of government employment into governmental functions and proprietary functions and of the many ramifications that this dichotomous treatment presents in the handling of concerted activities, collective bargaining, and strikes by government employees to wrest concessions in compensation, fringe benefits, hiring and firing, and other terms and conditions of employment.

The workers in the respondent institutions have not directly petitioned the heads of their respective offices nor their representatives in the Batasang Pambansa. They have acted through a labor federation and its affiliated unions. In other words, the workers and employees of these state firms, college, and university are taking collective action through a labor federation which uses the bargaining power of organized labor to secure increased compensation for its members.

Under the present state of the law and pursuant to the express language of the Constitution, this resort to concerted activity with the ever present threat of a strike can no longer be allowed.

The general rule in the past and up to the present is that "the terms and conditions of employment in the Government, including any political subdivision or instrumentality thereof are governed by law" (Section 11, the Industrial Peace Act, R.A. No. 875, as amended and Article 277, the Labor Code, P.D. No. 442, as amended). Since the terms and conditions of government employment are fixed by law, government workers cannot use the same weapons employed by workers in the private sector to secure concessions from their employers. The principle behind labor unionism in private industry is that industrial peace cannot be secured through compulsion by law. Relations between private employers and their employees rest on an essentially voluntary basis. Subject to the minimum requirements of wage laws and other labor and welfare

legislation, the terms and conditions of employment in the unionized private sector are settled through the process of collective bargaining. In government employment, however, it is the legislature and, where properly given delegated power, the administrative heads of government which fix the terms and conditions of employment. And this is effected through statutes or administrative circulars, rules, and regulations, not through collective bargaining agreements.

At the same time, the old Industrial Peace Act excepted employees and workers in proprietary functions of government from the above compulsion of law. Thus, in the past, government employees performing proprietary functions could belong to labor organizations imposing the obligation to join in strikes or engage in other concerted action. (Section 11, R.A. 875, as amended). They could and they did engage in concerted activities and various strikes against government owned and controlled corporations and other government institutions discharging proprietary functions. Among the institutions as falling under the exception in Section 11 of the Industrial Peace Act were respondents Government Service Insurance System (GSISEA v. Alvendia, 108 Phil. 505) and Social Security System (SSSEA v. Soriano, 7 SCRA 1016). And this Court has supported labor completely in the various strikes and concerted activities in firms and agencies discharging proprietary functions because the Constitution and the laws allowed these activities.

The exception, however belongs to the past.

The petitioners state in their counter comment filed July 23, 1982 that the 1973 Constitution is categorical about the grant of the rights to self- organization and collective bargaining to all workers and that no amount of stretched interpretation of lesser laws like the Labor Code and the Civil Service Act can overturn the clear message of the Constitution with respect to these rights to self-organization and collective bargaining.

These statements of the petitioners are error insofar as government workers are now concerned.

Under the present Constitution, govemment-owned or controlled corporations are specifically mentioned as embraced by the civil service. (Section 1, Article XII-B, Constitution). The inclusion of the clause "including every government owned or controlled corporation" in the 1973 amendments to the Constitution was a deliberate amendment for an express purpose. There may be those who disagree with the intent of the framers of the amendment but because it is fundamental law, we are all bound by it. The amendment was intended to correct the situation where more favored employees of the government could enjoy the benefits of two worlds. They were protected by the laws governing government employment. They could also engage in collective bargaining and join in strikes to secure higher wages

Page 16: Liberty of Abode and Travel to Freedom of Association

COMPILATION OF CASE DIGEST CONSTI 2: LIBERTY OF ABODE AND TRAVEL AND FREEDOM OF ASSOCIATION ISV Notes and fringe benefits which equally hardworking employees engaged in government functions could only envy but not enjoy.

Presidential Decree No. 807, the Civil Service Decree of the Philippines has implemented the 1973 Constitutional amendment. It is categorical about the inclusion of personnel of government-owned or controlled corporations in the civil service and their being subject to civil service requirements:

SECTION 56. Government- owned or Controlled Corporations Personnel.—All permanent personnel of government- owned or controlled corporations whose positions are now embraced in the civil service shall continue in the service until they have been given a chance to qualify in an appropriate examination, but in the meantime, those who do not possess the appropriate civil service eligibility shall not be promoted until they qualify in an appropriate civil service examination. Services of temporary personnel ma be y terminated any time.

Personnel of government-owned or controlled corporations are now part of the civil service. It would not be fair to allow them to engage in concerted activities to wring higher salaries or fringe benefits from Government even as other civil service personnel such as the hundreds of thousands of public school teachers, soldiers, policemen, health personnel, and other government workers are denied the right to engage in similar activities.

To say that the words "all employers" in P.D. No. 851 includes the Government and all its agencies, instrumentalities, and government-owned or controlled corporations would also result in nightmarish budgetary problems.

For instance, the Supreme Court is trying its best to alleviate the financial difficulties of courts, judges, and court personnel in the entire country but it can do so only within the limits of budgetary appropriations. Public school teachers have been resorting to what was formerly unthinkable, to mass leaves and demonstrations, to get not a 13th-month pay but promised increases in basic salaries and small allowances for school uniforms. The budget of the Ministry of Education, Culture and Sports has to be supplemented every now and then for this purpose. The point is, salaries and fringe benefits of those embraced by the civil service are fixed by law. Any increases must come from law, from appropriations or savings under the law, and not from concerted activity.

The Government Corporate Counsel, Justice Manuel Lazaro, in his consolidated comment * for respondents

GSIS, MWSS, and PVTA gives the background of the amendment which includes every government-owned or controlled corporation in the embrace of the civil service:

Records of the 1971 Constitutional Convention show that in the deliberations held relative to what is now Section 1(1) Article XII-B, supra the issue of the inclusion of government-owned or controlled corporations figured prominently.

The late delegate Roberto S. Oca, a recognized labor leader, vehemently objected to the inclusion of government-owned or controlled corporations in the Civil Service. He argued that such inclusion would put asunder the right of workers in government corporations, recognized in jurisprudence under the 1935 Constitution, to form and join labor unions for purposes of collective bargaining with their employers in the same manner as in the private section (see: records of 1971 Constitutional Convention).

In contrast, other labor experts and delegates to the 1971 Constitutional Convention enlightened the members of the Committee on Labor on the divergent situation of government workers under the 1935 Constitution, and called for its rectification. Thus, in a Position Paper dated November-22, 1971, submitted to the Committee on Labor, 1971 Constitutional Convention, then Acting Commissioner of Civil Service Epi Rev Pangramuyen declared:

It is the stand, therefore, of this Commission that by reason of the nature of the public employer and the peculiar character of the public service, it must necessarily regard the right to strike given to unions in private industry as not applying to public employees and civil service employees. It has been stated that the Government, in contrast to the private employer, protects the interests of all people in the public service, and

Page 17: Liberty of Abode and Travel to Freedom of Association

COMPILATION OF CASE DIGEST CONSTI 2: LIBERTY OF ABODE AND TRAVEL AND FREEDOM OF ASSOCIATION ISV Notes that accordingly, such conflicting interests as are present in private labor relations could not exist in the relations between government and those whom they employ.

Moreover, determination of employment conditions as well as supervision of the management of the public service is in the hands of legislative bodies. It is further emphasized that government agencies in the performance of their duties have a right to demand undivided allegiance from their workers and must always maintain a pronounced esprit de corps or firm discipline among their staff members. It would be highly incompatible with these requirements of the public service, if personnel took orders from union leaders or put solidarity with members of the working class above solidarity with the Government. This would be inimical to the public interest.

Moreover, it is asserted that public employees by joining labor unions may be compelled to support objectives which are political in nature and thus jeopardize the fundamental principle that the governmental machinery must be impartial and non-political in the sense of party politics.' (see: Records of 1971 Constitutional Convention).

Similarly, Delegate Leandro P. Garcia, expressing support for the inclusion of

government-owned or controlled corporations in the Civil Service, argued:

It is meretricious to contend that because Govermnent owned or controlled corporations yield profits, their employees are entitled to better wages and fringe benefits than employees of Government other than Government- owned and controlled cor orations which are not making profits. There is no gainsaying the fact that the capital they use is the people's (see Records of the 1971 Constitutional Convention).

Summarizing the deliberations of the 1971 Constitutional Convention on the inclusion of Government owned or controlled corporations, Dean Joaquin G. Bernas, SJ., of the Ateneo de Manila University Professional School of Law, stated that government-owned corporations came under attack as milking cows of a privileged few enjoying salaries far higher than their counterparts in the various branches of government, while the capital of these corporations belongs to the Government and government money is pumped into them whenever on the brink of disaster, and they should therefore come under the strick surveillance of the Civil Service System. (Bernas, The 1973 Philippine Constitution, Notes and Cases, 1974 ed., p. 524).

The Government Corporate Counsel cites the precedent setting decision in Agricultural- Credit and Cooperative Financing Administration (ACCFA v. Confederation of Unions in Government Corporations and Offtces CUGCO et al., 30 SCRA 649) as giving the rationale for coverage of government-owned or controlled corporations by the civil service. We stated ACCFA v. CUGCO that:

... The ACA is a government office or agency engaged in governmental, not proprietary functions. These functions may not be strictly what President Wilson described as "constituent" (as distinguished from 'ministrant'), [Bacani

Page 18: Liberty of Abode and Travel to Freedom of Association

COMPILATION OF CASE DIGEST CONSTI 2: LIBERTY OF ABODE AND TRAVEL AND FREEDOM OF ASSOCIATION ISV Notes vs. National Coconut Corporation, G.R. No. L-9657, Nov. 29,1956, 53 O.G. p. 2800] such as those relating to the maintenance of peace and the prevention of crime, those regulating property and property rights, those relating to the administration of justice and the determination of political duties of citizens, and those relating to national defense and foreign relations. Under this traditional classification, such constituent functions are exercised by the State as attributes of sovereignty, and not merely to promote the welfare, progress and prosperity of the people these latter functions being ministrant, the exercise of which is optional on the part of the government.

The growing complexities of modern society, however, have rendered this traditional classification of the functions of government quite unrealistic, not to say obsolete. The areas which used to be left to private enterprise and initiative and which the government was called upon to enter optionally, and only "because it was better equipped to administer for the public welfare than is any private individual or group of individuals," (Malcolm, The Government of the Philippines, pp. 19-20; Bacani vs. National Coconut Corporation, supra) continue to lose their well- defined boundaries and to be absorbed within activities that the government must undertake in its sovereign capacity if it is to meet the increasing social challenges of the times. Here as almost everywhere else the tendency is undoubtedly towards a greater socialization of economic forces, Here of course this development was envisioned, indeed adopted as a national policy, by the Constitution itself in its declaration of principle concerning the promotion of social justice.

Chief Justice Fernando, then an Associate Justice of this Court, observed in a concurring opinion that the traditional classification into constituent and ministrant functions reflects the primacy at that time of the now discredited and repudiated laissez faire concept carried over into government. He stated:

The influence exerted by American constitutional doctrines unavoidable when the Philippines was still under American rule notwithstanding, an influence that has not altogether vanished even after independence,

the laissez faire principle never found fun acceptance in this jurisdiction, even during the period of its full flowering in the United States. Moreover, to erase any doubts, the Constitutional Convention saw to it that our fundamental law embodies a policy of the responsibility thrust on government to cope with social and economic problems and an earnest and sincere commitment to the promotion of the general welfare through state action. It would thus follow that the force of any legal objection to regulatory measures adversely affecting property rights or to statutes organizing public corporations that may engage in competition with private enterprise has been blunted. Unless there be a clear showing of any invasion of rights guaranteed by the Constitution, their validity is a foregone conclusion. No fear need be entertained that thereby spheres hitherto deemed outside government domain have been encroached upon. With our explicit disavowal of the 'constituent-ministrant' test, the ghost of the laissez-faire concept no longer stalks the juridical stage."

Our dismissal of this petiti/n should not, by any means, be interpreted to imply that workers in government-owned and controlled corporations or in state colleges and universities may not enjoy freedom of association. The workers whom the petitioners purport to represent have the right, which may not be abridged, to form associations or societies for purposes not contrary to law. (Constitution, Article IV, Section 7). This is a right which share with all public officers and employees and, in fact, by everybody living in this country. But they may not join associations which impose the obligation to engage in concerted activities in order to get salaries, fringe benefits, and other emoluments higher than or different frm that provided by law and regulation.�

The very Labor Code, P.D. No. 442 as amended,, which governs the registration and provides for the rights of legitimate labor organizations states:

ART. 277. Government employees.— The terms and conditions of employment of all government employees, including employees of government-owned and controlled corporations, shall be governed by the Civil Service Law, rules and regulations. Their salaries shall be standardized by the National Assembly as provided for in the new constitution. However, there shall be no reduction of existing wages, benefits, and other terms and conditions

Page 19: Liberty of Abode and Travel to Freedom of Association

COMPILATION OF CASE DIGEST CONSTI 2: LIBERTY OF ABODE AND TRAVEL AND FREEDOM OF ASSOCIATION ISV Notes of employment being enjoyed by them at the time of the adoption of this code.

Section 6, Article XII-B of the Constitution gives added reasons why the government employees represented by the petitioners cannot expect treatment in matters of salaries different from that extended to all others government personnel. The provision states:

SEC. 6. The National Assembly shall provide for the standardization of compensation of government officials and employees, including those in government-owned or controlled corporations, taking into account the nature of the responsibilities pertaining to, and the qualifications required for the positions concerned.

It is the legislature or, in proper cases, the administrative heads of government and not the collective bargaining process nor the concessions wrung by labor unions from management that determine how much the workers in government-owned or controlled corporations may receive in terms of salaries, 13th month pay, and other conditions or terms of employment. There are government institutions which can afford to pay two weeks, three weeks, or even 13th-month salaries to their personnel from their budgetary appropriations. However, these payments must be pursuant to law or regulation. Presidential Decree No. 985 as amended provides:

xxx xxx xxx

SEC. 2. Declaration of Policy.— It is hereby declared to be the policy, of the national government to provide equal pay for substantially, equal work and to base differences in pay upon substantive differences in duties and responsibilities, and qualification requirements of the positions. In determining rates of pay, due regard shall be given to, among others, prevailing rates in private industry for comparable work. For this purpose, there is hereby established a system of compensation standardization and position classification in the national government for all departments, bureaus, agencies, and officers including government-owned or controlled corporations and financial institutions: Provided, That notwithstanding a standardized salary system established for all employees, additional financial incentives may be established by government corporations and financial institutions for their employees to be supported fully from

their corporate funds and for such technical positions as may be approved by the President in critical government agencies.

The Solicitor-General correctly points out that to interpret P.D. No. 851 as including government employees would upset the compensation levels of government employees in violation of those fixed according to P.D. No. 985.

Here as in other countries, government salaries and wages have always been lower than salaries, wages, and bonuses in the private sector. However, civil servants have no cause for despair. Service in the government may at times be a sacrifice but it is also a welcome privilege. Apart from the emotional and psychic satisfactions, there are various material advantages. The security of tenure guaranteed to those in the civil service by the Constitution and statutes, the knowledge that one is working for the most stable of employers and not for private persons, the merit system in appointments and promotions, the scheme of vacation, sick, and maternity leave privileges, and the prestige and dignity associated with public office are only a few of the joys of government employment.

Section 3 of the Rules and Regulations Implementing Presidential Decree No. 851 is, therefore, a correct interpretation of the decree. It has been implemented and enforced from December 22, 1975 to the present, The petitioners have shown no valid reason why it should be nullified because of their petition filed six and a half years after the issuance and implementation of the rule.

WHEREFORE, the petition is hereby DISMISSED for lack of merit.

SO ORDERED.

6. GSIS vs Mga Kapisanan ng mga Manggagawa sa GSIS

In this petition for review on certiorari under Rule 45 of the Rules of Court, the Government Service Insurance System (GSIS) and its President and General Manager Winston F. Garcia (Garcia, for short) assail and seek to nullify the Decision1 dated June 16, 2005 of the Court of Appeals (CA) in CA-G.R. SP No. 87220, as reiterated in its Resolution2 of October 18, 2005 denying Garcia's motion for reconsideration.

The recourse is cast against the following setting:

A four-day October 2004 concerted demonstration, rallies and en masse walkout waged/held in front of the

Page 20: Liberty of Abode and Travel to Freedom of Association

COMPILATION OF CASE DIGEST CONSTI 2: LIBERTY OF ABODE AND TRAVEL AND FREEDOM OF ASSOCIATION ISV Notes GSIS main office in Roxas Boulevard, Pasay City, started it all. Forming a huge part of the October 4 to October 7, 2004 mass action participants were GSIS personnel, among them members of the herein respondent Kapisanan Ng Mga Manggagawa sa GSIS ("KMG" or the "Union"), a public sector union of GSIS rank-and-file employees. Contingents from other government agencies joined causes with the GSIS group. The mass action's target appeared to have been herein petitioner Garcia and his management style. While the Mayor of Pasay City allegedly issued a rally permit, the absence of the participating GSIS employees was not covered by a prior approved leave.3

On or about October 10, 2004, the manager of the GSIS Investigating Unit issued a memorandum directing 131 union and non-union members to show cause why they should not be charged administratively for their participation in said rally. In reaction, KMG's counsel, Atty. Manuel Molina, sought reconsideration of said directive on the ground, among others, that the subject employees resumed work on October 8, 2004 in obedience to the return-to-work order thus issued. The plea for reconsideration was, however, effectively denied by the filing, on October 25, 2004, of administrative charges against some 110 KMG members for grave misconduct and conduct prejudicial to the best interest of the service.4

What happened next is summarized by the CA in its challenged decision of June 16, 2005, albeit the herein petitioners would except from some of the details of the appellate court's narration:

Ignoring said formal charges, KMG, thru its President, Albert Velasco, commenced the instant suit on November 2, 2004, with the filing of the Petition for Prohibition at bench. On the ground that its members should not be made to explain why they supported their union's cause, petitioner [KMG] faulted respondent [Garcia] with blatant disregard of Civil Service Resolution No. 021316, otherwise known as the Guidelines for Prohibited Mass Action, Section 10 of which exhorts government agencies to "harness all means within their capacity to accord due regard and attention to employees' grievances and facilitate their speedy and amicable disposition through the use of grievance machinery or any other modes of settlement sanctioned by law and existing civil service rules." Two supplements to the foregoing petition were eventually filed by KMG. The first, … apprised [the CA] of the supposed fact that its Speaker, Atty. Molina, had been placed under preventive suspension for 90 days and that the formal charges thus filed will not only deprive its members of the privileges and benefits due them but will also disqualify them from promotion, step increment adjustments and receipt of monetary benefits, including their 13th

month pay and Christmas bonuses. The second, xxx manifested that, on December 17, 2004, respondent [Garcia] served a spate of additional formal charges against 230 of KMG's members for their participation in the aforesaid grievance demonstrations.

In his December 14, 2004 comment to the foregoing petition, respondent [Garcia] averred that the case at bench was filed by an unauthorized representative in view of the fact that Albert Velasco had already been dropped from the GSIS rolls and, by said token, had ceased to be a member – much less the President – of KMG. Invoking the rule against forum shopping, respondent [Garcia] called [the CA's] attention to the supposed fact that the allegations in the subject petition merely duplicated those already set forth in two petitions for certiorari and prohibition earlier filed by Albert Velasco …. Because said petitions are, in point of fact, pending before this court as CA-G.R. SP Nos. 86130 and 86365, respondent [Garcia] prayed for the dismissal of the petition at bench ….5 (Words in bracket added.)

It appears that pending resolution by the CA of the KMG petition for prohibition in this case, the GSIS management proceeded with the investigation of the administrative cases filed. As represented in a pleading before the CA, as of May 18, 2005, two hundred seven (207) out of the two hundred seventy eight (278) cases filed had been resolved, resulting in the exoneration of twenty (20) respondent-employees, the reprimand of one hundred eighty two (182) and the suspension for one month of five (5).6

On June 16, 2005, the CA rendered the herein assailed decision7 holding that Garcia's "filing of administrative charges against 361 of [KMG's] members is tantamount to grave abuse of discretion which may be the proper subject of the writ of prohibition." Dispositively, the decision reads:

WHEREFORE, premises considered, the petition [of KMG] is GRANTED and respondent [Winston F. Garcia] is hereby PERPETUALLY ENJOINED from implementing the issued formal charges and from issuing other formal charges arising from the same facts and events.

SO ORDERED. (Emphasis in the original)

Unable to accept the above ruling and the purported speculative factual and erroneous legal premises holding it together, petitioner Garcia sought reconsideration. In its equally assailed Resolution8 of October 18, 2005, however, the appellate court denied reconsideration of its decision.

Page 21: Liberty of Abode and Travel to Freedom of Association

COMPILATION OF CASE DIGEST CONSTI 2: LIBERTY OF ABODE AND TRAVEL AND FREEDOM OF ASSOCIATION ISV Notes Hence, this recourse by the petitioners ascribing serious errors on the appellate court in granting the petition for prohibition absent an instance of grave abuse of authority on their part.

We resolve to GRANT the petition.

It should be stressed right off that the civil service encompasses all branches and agencies of the Government, including government-owned or controlled corporations (GOCCs) with original charters, like the GSIS,9 or those created by special law.10 As such, employees of covered GOCCs are part of the civil service system and are subject to circulars, rules and regulations issued by the Civil Service Commission (CSC) on discipline, attendance and general terms/conditions of employment, inclusive of matters involving self-organization, strikes, demonstrations and like concerted actions. In fact, policies established on public sector unionism and rules issued on mass action have been noted and cited by the Court in at least a case.11 Among these issuances is Executive Order (EO) No. 180, series of 1987, providing guidelines for the exercise of the right to organize of government employees. Relevant also is CSC Resolution No. 021316 which provides rules on prohibited concerted mass actions in the public sector.

There is hardly any dispute about the formal charges against the 278 affected GSIS employees – a mix of KMG union and non-union members - having arose from their having gone on unauthorized leave of absence (AWOL) for at least a day or two in the October 4 to 7, 2004 stretch to join the ranks of the demonstrators /rallyists at that time. As stated in each of the formal charges, the employee's act of attending, joining, participating and taking part in the strike/rally is a transgression of the rules on strike in the public sector. The question that immediately comes to the fore, therefore, is whether or not the mass action staged by or participated in by said GSIS employees partook of a strike or prohibited concerted mass action. If in the affirmative, then the denounced filing of the administrative charges would be prima facie tenable, inasmuch as engaging in mass actions resulting in work stoppage or service disruption constitutes, in the minimum, the punishable offense of acting prejudicial to the best interest of the service.12 If in the negative, then such filing would indeed smack of arbitrariness and justify the issuance of a corrective or preventive writ.

Petitioners assert that the filing of the formal charges are but a natural consequence of the service-disrupting rallies and demonstrations staged during office hours by the absenting GSIS employees, there being appropriate issuances outlawing such kinds of mass action. On the other hand, the CA, agreeing with the respondent's argument, assumed the view and held that the organized demonstrating employees did nothing more than air their grievances in the exercise of their "broader rights of free

expression"13 and are, therefore, not amenable to administrative sanctions. For perspective, following is what the CA said:

Although the filing of administrative charges against [respondent KMG's] members is well within [petitioner Garcia's] official [disciplinary] prerogatives, [his] exercise of the power vested under Section 45 of Republic Act No. 8291 was tainted with arbitrariness and vindictiveness against which prohibition was sought by [respondent]. xxx the fact that the subject mass demonstrations were directed against [Garcia's] supposed mismanagement of the financial resources of the GSIS, by and of itself, renders the filing of administrative charges against [KMG's] member suspect. More significantly, we find the gravity of the offenses and the sheer number of persons … charged administratively to be, at the very least, antithetical to the best interest of the service….

It matters little that, instead of the 361 alleged by petitioner, only 278 charges were actually filed [and] in the meantime, disposed of and of the said number, 20 resulted to exoneration, 182 to reprimand and 5 to the imposition of a penalty of one month suspension. Irrespective of their outcome, the severe penalties prescribed for the offense with which petitioner's members were charged, to our mind, bespeak of bellicose and castigatory reaction …. The fact that most of the employees [Garcia] administratively charged were eventually meted with what appears to be a virtual slap on the wrist even makes us wonder why respondent even bothered to file said charges at all. xxx.

Alongside the consequences of the right of government employees to form, join or assist employees organization, we have already mentioned how the broader rights of free expression cast its long shadow over the case. xxx we find [petitioner Garcia's] assailed acts, on the whole, anathema to said right which has been aptly characterized as preferred, one which stands on a higher level than substantive economic and other liberties, the matrix of other important rights of our people. xxx.14 (Underscoring and words in bracket added; citations omitted.)

While its decision and resolution do not explicitly say so, the CA equated the right to form associations with the right to engage in strike and similar activities available to workers in the private sector. In the concrete, the appellate court concluded that inasmuch as GSIS employees are not barred from forming, joining or assisting employees' organization, petitioner Garcia could not validly initiate charges against GSIS

Page 22: Liberty of Abode and Travel to Freedom of Association

COMPILATION OF CASE DIGEST CONSTI 2: LIBERTY OF ABODE AND TRAVEL AND FREEDOM OF ASSOCIATION ISV Notes employees waging or joining rallies and demonstrations notwithstanding the service-disruptive effect of such mass action. Citing what Justice Isagani Cruz said in Manila Public School Teachers Association [MPSTA] v. Laguio, Jr.,15 the appellate court declared:

It is already evident from the aforesaid provisions of Resolution No. 021316 that employees of the GSIS are not among those specifically barred from forming, joining or assisting employees organization such as [KMG]. If only for this ineluctable fact, the merit of the petition at bench is readily discernible.16

We are unable to lend concurrence to the above CA posture. For, let alone the fact that it ignores what the Court has uniformly held all along, the appellate court's position is contrary to what Section 4 in relation to Section 5 of CSC Resolution No. 02131617 provides. Besides, the appellate court's invocation of Justice Cruz's opinion inMPSTA is clearly off-tangent, the good Justice's opinion thereat being a dissent. It may be, as the appellate court urged¸ that the freedom of expression and assembly and the right to petition the government for a redress of grievances stand on a level higher than economic and other liberties. Any suggestion, however, about these rights as including the right on the part of government personnel to strike ought to be, as it has been, trashed. We have made this abundantly clear in our past determinations. For instance, in Alliance of Government Workers v. Minister of Labor and Employment,18 a case decided under the aegis of the 1973 Constitution, an en banc Court declared that it would be unfair to allow employees of government corporations to resort to concerted activity with the ever present threat of a strike to wring benefits from Government. Then came the 1987 Constitution expressly guaranteeing, for the first time, the right of government personnel to self-organization19 to complement the provision according workers the right to engage in "peaceful concerted activities, including the right to strike in accordance with law."20

It was against the backdrop of the aforesaid provisions of the 1987 Constitution that the Court resolvedBangalisan v. Court of Appeals.21 In it, we held, citing MPSTA v. Laguio, Jr.,22 that employees in the public service may not engage in strikes or in concerted and unauthorized stoppage of work; that the right of government employees to organize is limited to the formation of unions or associations, without including the right to strike.

Jacinto v. Court of Appeals23 came next and there we explained:

Specifically, the right of civil servants to organize themselves was positively recognized in Association of Court of Appeals Employees vs. Ferrer-Caleja. But, as in the exercise of the

rights of free expression and of assembly, there are standards for allowable limitations such as the legitimacy of the purpose of the association, [and] the overriding considerations of national security . . . .

As regards the right to strike, the Constitution itself qualifies its exercise with the provision "in accordance with law." This is a clear manifestation that the state may, by law, regulate the use of this right, or even deny certain sectors such right. Executive Order 180 which provides guidelines for the exercise of the right of government workers to organize, for instance, implicitly endorsed an earlier CSC circular which "enjoins under pain of administrative sanctions, all government officers and employees from staging strikes, demonstrations, mass leaves, walkouts and other forms of mass action which will result in temporary stoppage or disruption of public service" by stating that the Civil Service law and rules governing concerted activities and strikes in government service shall be observed. (Emphasis and words in bracket added; citations omitted)

And in the fairly recent case of Gesite v. Court of Appeals,24 the Court defined the limits of the right of government employees to organize in the following wise:

It is relevant to state at this point that the settled rule in this jurisdiction is that employees in the public service may not engage in strikes, mass leaves, walkouts, and other forms of mass action that will lead in the temporary stoppage or disruption of public service. The right of government employees to organize is limited to the formation of unions or associations only, without including the right to strike,

adding that public employees going on disruptive unauthorized absences to join concerted mass actions may be held liable for conduct prejudicial to the best interest of the service.

Significantly, 1986 Constitutional Commission member Eulogio Lerum, answering in the negative the poser of whether or not the right of government employees to self-organization also includes the right to strike, stated:

When we proposed this amendment providing for self organization of government employees, it does not mean that because they have the right to organize, they have also the right to strike. That is a different matter. xxx25

With the view we take of the events that transpired on October 4-7, 2004, what respondent's members launched or participated in during that time partook of a

Page 23: Liberty of Abode and Travel to Freedom of Association

COMPILATION OF CASE DIGEST CONSTI 2: LIBERTY OF ABODE AND TRAVEL AND FREEDOM OF ASSOCIATION ISV Notes strike or, what contextually amounts to the same thing, a prohibited concerted activity. The phrase "prohibited concerted activity" refers to any collective activity undertaken by government employees, by themselves or through their employees' organization, with the intent of effecting work stoppage or service disruption in order to realize their demands or force concessions, economic or otherwise; it includes mass leaves, walkouts, pickets and acts of similar nature.26 Indeed, for four straight days, participating KMG members and other GSIS employees staged a walk out and waged or participated in a mass protest or demonstration right at the very doorstep of the GSIS main office building. The record of attendance27 for the period material shows that, on the first day of the protest, 851 employees, or forty eight per cent (48%) of the total number of employees in the main office (1,756) took to the streets during office hours, from 6 a.m. to 2 p.m.,28leaving the other employees to fend for themselves in an office where a host of transactions take place every business day. On the second day, 707 employees left their respective work stations, while 538 participated in the mass action on the third day. A smaller number, i.e., 306 employees, but by no means an insignificant few, joined the fourth day activity.

To say that there was no work disruption or that the delivery of services remained at the usual level of efficiency at the GSIS main office during those four (4) days of massive walkouts and wholesale absences would be to understate things. And to place the erring employees beyond the reach of administrative accountability would be to trivialize the civil service rules, not to mention the compelling spirit of professionalism exacted of civil servants by the Code of Conduct and Ethical Standards for Public Officials and Employees. 29

The appellate court made specific reference to the "parliament of the streets," obviously to lend concurrence to respondent's pretension that the gathering of GSIS employees on October 4-7, 2004 was an "assembly of citizens" out only to air grievances, not a striking crowd. According to the respondent, a strike presupposes a mass action undertaken to press for some economic demands or secure additional material employment benefits.

We are not convinced.

In whatever name respondent desires to call the four-day mass action in October 2004, the stubborn fact remains that the erring employees, instead of exploring non-crippling activities during their free time, had taken a disruptive approach to attain whatever it was they were specifically after. As events evolved, they assembled in front of the GSIS main office building during office hours and staged rallies and protests, and even tried to convince others to join their cause, thus provoking work stoppage and service-delivery disruption, the very evil sought to be forestalled by the prohibition against strikes by government personnel.30

The Court can concede hypothetically that the protest rally and gathering in question did not involve some specific material demand. But then the absence of such economic-related demand, even if true, did not, under the premises, make such mass action less of a prohibited concerted activity. For, as articulated earlier, any collective activity undertaken by government employees with the intent of effecting work stoppage or service disruption in order to realize their demands or force concessions, economic or otherwise, is a prohibited concerted mass action31 and doubtless actionable administratively. Bangalisan even went further to say the following: "[i]n the absence of statute, public employees do not have the right to engage in concerted work stoppages for any purpose."

To petitioner Garcia, as President and General Manager of GSIS, rests the authority and responsibility, under Section 45 of Republic Act No. 8291, the GSIS Act of 1997, to remove, suspend or otherwise discipline GSIS personnel for cause.32 At bottom then, petitioner Garcia, by filing or causing the filing of administrative charges against the absenting participants of the October 4-7, 2004 mass action, merely performed a duty expected of him and enjoined by law. Regardless of the mood petitioner Garcia was in when he signed the charge sheet, his act can easily be sustained as legally correct and doubtless within his jurisdiction.

It bears to reiterate at this point that the GSIS employees concerned were proceeded against - and eventually either exonerated, reprimanded or meted a one-month suspension, as the case may be - not for the exercise of their right to assemble peacefully and to petition for redress of grievance, but for engaging in what appeared to be a prohibited concerted activity. Respondent no less admitted that its members and other GSIS employees might have disrupted public service.33

To be sure, arbitrariness and whimsical exercise of power or, in fine, grave abuse of discretion on the part of petitioner Garcia cannot be simplistically inferred from the sheer number of those charged as well as the gravity or the dire consequences of the charge of grave misconduct and conduct prejudicial to the best interest of the service, as the appellate court made it to appear. The principle of accountability demands that every erring government employee be made answerable for any malfeasance or misfeasance committed. And lest it be overlooked, the mere filing of formal administrative case, regardless of the gravity of the offense charged, does not overcome the presumptive innocence of the persons complained of nor does it shift the burden of evidence to prove guilt of an administrative offense from the complainant.

Moreover, the Court invites attention to its holding in MPSTA v. Laguio, Jr., a case involving over 800 public school teachers who took part in mass actions for which the then Secretary of Education filed

Page 24: Liberty of Abode and Travel to Freedom of Association

COMPILATION OF CASE DIGEST CONSTI 2: LIBERTY OF ABODE AND TRAVEL AND FREEDOM OF ASSOCIATION ISV Notes administrative complaints on assorted charges, such as gross misconduct. Of those charged, 650 were dismissed and 195 suspended for at least six (6) months The Court, however, did not consider the element of number of respondents thereat and/or the dire consequences of the charge/s as fatally vitiating or beclouding the bona fidesof the Secretary of Education's challenged action. Then as now, the Court finds the filing of charges against a large number of persons and/or the likelihood that they will be suspended or, worse, dismissed from the service for the offense as indicating a strong and clear case of grave abuse of authority to justify the issuance of a writ of prohibition.

The appellate court faulted petitioner Garcia for not first taping existing grievance machinery and other modes of settlement agreed upon in the GSIS-KMG Collective Negotiations Agreement (CAN) before going full steam ahead with his formal charges.34

The Court can plausibly accord cogency to the CA's angle on grievance procedure but for the fact that it conveniently disregarded what appears to be the more relevant provision of the CNA. We refer to Article VI which reads:

The GSIS Management and the KMG have mutually agreed to promote the principle of shared responsibility … on all matters and decisions affecting the rights, benefits and interests of all GSIS employees …. Accordingly, … the parties also mutually agree that the KMG shall not declare a strike nor stage any concerted action which will disrupt public service and the GSIS management shall not lockoutemployees who are members of the KMG during the term of this agreement. GSIS Management shall also respect the rights of the employees to air their sentiments through peaceful concerted activities during allowable hours, subject to reasonable office rules ....35 (Underscoring added)

If the finger of blame, therefore, is to be pointed at someone for non-exhaustion of less confrontational remedies, it should be at the respondent union for spearheading a concerted mass action without resorting to available settlement mechanism. As it were, it was KMG, under Atty. Alberto Velasco, which opened fire first. That none of the parties bothered to avail of the grievance procedures under the GSIS-KMG CNA should not be taken against the GSIS. At best, both GSIS management and the Union should be considered as in pari delicto.

With the foregoing disquisitions, the Court finds it unnecessary to discuss at length the legal standing of Alberto Velasco to represent the herein respondent union and to initiate the underlying petition for prohibition. Suffice it to state that Velasco, per Joint

Resolution No. 04-10-01 approved on October 5, 2004 by the KMG Joint Executive-Legislative Assembly, had ceased to be member, let alone president, of the KMG, having previously been dropped from the rolls of GSIS employees.36 While the dropping from the rolls is alleged to have been the subject of a CA-issued temporary restraining order (TRO), the injunction came after Atty. Velasco had in fact been separated from the service and it appears that the TRO had already expired.

As a final consideration, the Court notes or reiterates the following relevant incidents surrounding the disposition of the case below:

1. The CA had invoked as part of its ratio decidendi a dissenting opinion in MPSTA, even going to the extent of describing as "instructive and timely" a portion, when the majority opinion thereat, which the appellate court ignored, is the controlling jurisprudence.

2. The CA gave prominence to dispositions and rattled off holdings37 of the Court, which appropriately apply only to strikes in the private industry labor sector, and utilized the same as springboard to justify an inference of grave abuse of discretion. On the other hand, it only gave perfunctory treatment if not totally ignored jurisprudence that squarely dealt with strikes in the public sector, as if the right to strike given to unions in private corporations/entities is necessarily applicable to civil service employees.

3. As couched, the assailed CA decision perpetually bars respondent Garcia – and necessarily whoever succeeds him as GSIS President – not only from implementing the formal charges against GSIS employees who participated in the October 4 - 7, 2004 mass action but also from issuing other formal charges arising from the same events. The injunction was predicated on a finding that grave abuse of discretion attended the exercise of petitioner Garcia's disciplinary power vested him under Section 45 of RA 8291.38 At bottom then, the assailed decision struck down as a nullity, owing to the alleged attendant arbitrariness, not only acts that have already been done, but those yet to be done. In net effect, any formal charge arising from the October 4-7, 2004 incident is, under any and all circumstances, prejudged as necessarily tainted with arbitrariness to be slain at sight.

The absurdities and ironies easily deducible from the foregoing situations are not lost on the Court.

We close with the observation that the assailed decision and resolution, if allowed to remain undisturbed, would

Page 25: Liberty of Abode and Travel to Freedom of Association

COMPILATION OF CASE DIGEST CONSTI 2: LIBERTY OF ABODE AND TRAVEL AND FREEDOM OF ASSOCIATION ISV Notes likely pave the way to the legitimization of mass actions undertaken by civil servants, regardless of their deleterious effects on the interest of the public they have sworn to serve with loyalty and efficiency. Worse still, it would permit the emergence of a system where public sector workers are, as the petitioners aptly put it, "immune from the minimum reckoning for acts that [under settled jurisprudence] are concededly unlawful." This aberration would be intolerable.

WHEREFORE, the assailed Decision and Resolution of the Court of Appeals are REVERSED and SET ASIDE and the writ of prohibition issued by that court is NULLIFIED.

No Cost.

SO ORDERED.

Puno, J., Chairperson, Sandoval-Gutierrez, and Azcuna, JJ., concur.Corona, J., On Leave.